LSAT and Law School Admissions Forum

Get expert LSAT preparation and law school admissions advice from PowerScore Test Preparation.

 Administrator
PowerScore Staff
  • PowerScore Staff
  • Posts: 8917
  • Joined: Feb 02, 2011
|
#22883
Complete Question Explanation

Strengthen X—CE. The correct answer choice is (B)

The child psychologist quoted here discusses studies of young children’s reactions to violent video games. Based on the fact that the children have been found to act more aggressively after playing the games, the psychologist concludes that the violence in video games lead children to believe that violence is acceptable:

violence in video games → kids believe violence acceptable → kids behave more aggressively
Since the stimulus is followed by a Strengthen Except question, the four incorrect answer choices will strengthen the child psychologist’s argument, and the one correct choice will fail to strengthen the argument.

Answer choice (A): This answer choice strengthens the conclusion that there exists a causal link between video game play, and the belief that violence is acceptable. If children tend to be more accepting of aggressive behavior in others after playing violent video games, it would seem more likely that the causal relationship hypothesized in the stimulus is valid.

Answer choice (B): This is the correct answer choice, because it fails to strengthen the causal argument in the stimulus. The fact that many (a vague numerical term) children without the supposed cause still experience the same supposed effect does not bolster the psychologist’s argument. In fact, as we might recognize, one way to weaken a causal argument is to reference situations in which the hypothesized effect occurs in the absence of the supposed cause. Since this is the only answer choice which fails to strengthen the causal argument presented in the stimulus, it is the correct answer choice.

Answer choice (C): This answer choice strengthens the causal argument in the conclusion by ruling out one alternative cause: since those playing non-violent video games did not experience the same effects, it rules out the possibility that video games in general lead to the stated effect. The argument in the stimulus is that the children’s belief that violence is acceptable is caused by the violence in violent video games.

Answer choice (D): This choice, which basically states that older children are also more likely to believe violence to be acceptable after playing violent video games. If other types of children (older ones) experience the same effects, this strengthens the child psychologist’s assertion of a link between violence and such accepting beliefs.

Answer choice (E): This answer also strengthens the psychologist’s argument, by strengthening the link between the belief that violence is acceptable, and aggressive behavior. Since this answer choice does effectively strengthen the argument in the stimulus, it is an incorrect response to this Strengthen X question.
 ay514
  • Posts: 27
  • Joined: Aug 02, 2011
|
#1547
Hello,

I had a hard time figuring out why answer choice B was a better answer than choice E. To be honest, I thought both statements didn't do much to not strengthen the argument. This is one of the earlier questions of the section so I racked my brains trying to figure out if I was totally missing something here. Thanks in advance for your help!!
 Nikki Siclunov
PowerScore Staff
  • PowerScore Staff
  • Posts: 1362
  • Joined: Aug 02, 2011
|
#1562
The psychologist concludes that violence in video games leads young children to believe that aggressive behavior is acceptable (last sentence of the stimulus). The question stem asks us to identify the one answer choice that does NOT strengthen the argument.

Answer choice (E) strengthens the causal reasoning underlying the conclusion by offering an example in which an analogous cause and effect has been observed. After all, if young children tend to behave more aggressively immediately after being told that such behavior is acceptable, and some video games suggest that violent behavior is acceptable, then it is even more likely that the aggressive behavior observed among the children who just played such games is the result of playing video games. Because this is an example in which an analogous cause and effect has been found to occur, answer choice (E) strengthens the argument and is therefore incorrect.

Answer choice (B) weakens the argument by showing that the effect can occur (finding aggressive behavior to be acceptable) without the cause (these children never played video games). Therefore, this is the correct answer choice.
 andriana.caban
  • Posts: 142
  • Joined: Jun 23, 2017
|
#36783
I'm confused about how we know that this stimulus has a casual relationship?
 Luke Haqq
PowerScore Staff
  • PowerScore Staff
  • Posts: 742
  • Joined: Apr 26, 2012
|
#36799
Hi Andriana,

There are a couple of causal claims in this, though they're implicit.
  • Some studies in which children have been observed before and after playing video games with violent content have shown that young children tend to behave more aggressively immediately after playing the games. This suggests that the violence in such video games leads young children to believe that aggressive behavior is acceptable.


In the first sentence, the causal relationship is between aggression and playing violent video games. It's the words "after playing' that the author is using to suggest there's a causal link--e.g., since event B consistently happens after event A, perhaps B caused A. In the next sentence, it's saying that there's a causal relationship between violent video games and children's beliefs that aggressive behavior is acceptable. The word "leads" indicates that the author is implicitly making a causal claim--i.e., A leads to B, is another way of saying A caused B.

Hope that helps!
 lunsandy
  • Posts: 61
  • Joined: Oct 14, 2017
|
#42448
Hi Powerscore,

I was hesitant about question D because it says "older children" and I thought our conclusion only refers to "young children." I thought answer D then was out of scope and thus did nothing to our argument (StrengthenX). Yes, I agree that B is the correct answer because it weakens the argument through effect w/o a cause. But can't we get rid of D for being out of scope?

Thanks a lot!
 Jennifer Janowsky
PowerScore Staff
  • PowerScore Staff
  • Posts: 90
  • Joined: Aug 20, 2017
|
#42469
lunsandy,

The fact that (D) talked about "older children" did make it a slightly out of scope answer choice for this stimulus. However, in Strengthen questions, these answer choices do not necessarily have to be eliminated so long as they help the argument. Test makers can bring in outside information in Strengthening and Weakening questions in order to support or hurt the conclusion provided.

Hope that answers your question!
 student987
  • Posts: 28
  • Joined: Apr 09, 2018
|
#48632
Hi Powerscore.

I'm posting a follow-up question to the previous response. Can you explain what it means for a “slightly out of scope” answer choice to strengthen or weaken an argument? I'm still confused as to what (D) has to do with "young children." If (D) had said just "children," I understand that though it's beyond the scope of the argument, it still helps the argument. But I don't get how "older children" helps.
 Francis O'Rourke
PowerScore Staff
  • PowerScore Staff
  • Posts: 471
  • Joined: Mar 10, 2017
|
#48772
Hi Student987!

I believe that Jennifer was trying to say that it was only slightly out of scope. That is, although the stimulus discuses young children and answer choice (D) discusses older children, the two groups are by commonsense standards somewhat similar.

Although answer choice (D) would have been much stronger had it provided evidence of children in general, it is fair to assume that these two similar groups may have similar reactions.

For example, if I gave you a study of the negative effects of alcohol on children aged 12 to 15, you would probably make some inferences about the likelihood of negative effects of alcohol on children below the age of 12. Even if that study were slightly out of scope compared to a study on younger children, it would likely give us some information about the group we care about.

This is basically what answer choice (D) is doing. Although it's scope is slightly off compared to the evidence we would hope for, the groups are similar enough that the evidence somewhat strengthens the psychologist's argument.

I hope this explanation helps, but let us know if you have any other questions! :)
 tfab
  • Posts: 5
  • Joined: Feb 17, 2020
|
#74136
For this I chose B, but I was curious about the wording for Answer "E." Could it be argued that since young children behave more aggressively immediately after being told that aggressive behavior is acceptable, that violent video games is similar to them being told that aggressive behavior is acceptable? They believe that it's acceptable because they see it in the video games. That's the way I interpreted it at least and led to me eliminating this answer choice. Thanks!

Get the most out of your LSAT Prep Plus subscription.

Analyze and track your performance with our Testing and Analytics Package.